LSAT and Law School Admissions Forum

Get expert LSAT preparation and law school admissions advice from PowerScore Test Preparation.

 Administrator
PowerScore Staff
  • PowerScore Staff
  • Posts: 8916
  • Joined: Feb 02, 2011
|
#40575
Complete Question Explanation
(The complete setup for this game can be found here: lsat/viewtopic.php?t=5718)

The correct answer choice is (E)

In a Cannot Be True, List question, immediately look to the Not Laws for an answer. In early questions in a game that approach will often yield the correct answer quite quickly. This question appears second-to-last, and perhaps not unexpectedly the Not Laws do not yield a correct answer. Thus, this question will require a deeper solution.

The correct answer will supply two movies that cannot jointly be shown on screen 1. Since none of the answer choices creates an obvious violation, you should expect that the groups will play a role, since grouping solutions are often more challenging for the typical test taker to identify. From a grouping perspective, the three movies not named in the correct answer will have to be shown on screens 2 and 3:
PT70 -Game_#3_#17_diagram 1.png
The problematic variables within screens 2 and 3 are H and M, which cannot be shown together from the fourth rule, and R and S, which have restrictions on screen 2 and screen 3, respectively. Because those are the problem variables within screens 2 and 3, you’d expect to see other variables in the correct answer choice (as they would force these variables onto screens 2 and 3).

Specifically, W was not mentioned above, so you would expect to see W in the correct answer. That limits our focus to answer choices (C), (D), and (E). You would not want H or M in the answer choice, because you want them forced together elsewhere. Answers (C) and (D) contain H and M, respectively, and are thus less attractive. That leaves answer choice (E), which includes W and S. Let’s examine this answer more closely.

If W and S are shown on screen 1, then H, M, and R must be shown on screens 2 and 3, collectively:
PT70 -Game_#3_#17_diagram 2.png
Because R cannot be shown on screen 2 due to the third rule, R must be shown on screen 3,. That then forces H and M to be shown on screen 2, which is a violation of the fourth rule. Thus, W and S cannot be shown together on screen 1, and answer choice (E) is correct.
You do not have the required permissions to view the files attached to this post.

Get the most out of your LSAT Prep Plus subscription.

Analyze and track your performance with our Testing and Analytics Package.